285. Nice and easy inequalities. For example, IMAC - 2009.
by Virgil Nicula, Jun 13, 2011, 1:42 PM
IMAC - 2009. Prove that for the sides of triangle
exists the inequality
.
Proof 1.
thus remaining to show that
, which is obviously.
Proof 2. Since
and
.
Can apply the Chebyshev's inequality so that
.
Similar proposed problem. Prove that
and
, where
and
.
Proof. I"ll use the well-known identity
. Thus,

. I"ll proceed analogously

.
Otherwise.
. Therefore,
.
Observe that
and

what is obviously. Therefore obtain that
. In conclusion and
.
PP2. Prove that
.
Proof. Denote
so that
. Observe that
and our inequality
becomes

, what is truly.
An easy and nice extension. Prove that
.
Proof. Denote
, where
. Prove easily that
.
Therefore,

.
PP3. Prove that for any point
which is interior to
there is the relation
.
Proof. Observe that
a.s.o. and
. Therefore,
.
PP4. Find the minimum value of
.
Proof. Denote
. Observe that

.
We have equality if and only if
, i.e.
, where
.
PP5. Let
be a triangle so that
. Prove that
with equality
iff
. Remark. If
, then our inequality becomes
with equality iff
.
Proof 1. If
, then the inequality
becomes
, what is true. We have equality iff
, i.e. 
. Suppose
and denote
, i.e.
, where
. Inequality
becomes 

, what is true. We have equality iff
, i.e.
.
Remark. If
, i.e.
, then
becomes
, with equality iff
.
Proof 2. By squaring both sides of the given inequality one obtains:
, which
is trivial by AM-GM inequality. Equality occurs iff

.
PP6. Let
be a point inside the
. Denote the distances
,
and
from
to the sides of
. Prove
that
, where R is the circumradius of the triangle. When does equality occur ? (Taiwan TST 2005)
Proof.
. So,
![$12RS=3\cdot a^2b^2c^2\le\sqrt [3]{abc}\cdot\sum a^2\implies$](//latex.artofproblemsolving.com/8/3/d/83d65e6ea4cf3a349a5730ebf1f43798557d789e.png)
.
PP7. Prove that
.
Proof 1. Minkowski's inequality :
.
Proof 2. Let
,
where
separates
,
and
.
Observe that
and our inequality is equivalently with
, what is truly. We have equality iff 
in
the edge
is the
-bisector in

. Indeed, if
, then the inequality
becomes
. Obtain equality
and in this case
.
Proof 3.


.
An easy extension.
with equality iff
.
Proof. Let
,
where
separates
,
and
, where
. Denote ![$m=2\cos\phi\in [0,2]$](//latex.artofproblemsolving.com/9/4/3/9431fe14c1d7c22b2fae6a41cedf222ddb575fdc.png)
Observe that
and
and our inequality is equivalently with
,
what is truly. We have equality
in
the edge
is the
-bisector in 

.
PP8. Let
be positive real numbers. Prove that
.
Proof.
.
An easy extension. Let
be positive real numbers. Prove that
for any
.
Proof.
for any
.
PP9. Let
be three positive numbers . Prove that
.
Proof. Denote
, i.e.
.
Observe that
.
Therefore,

.
In conclusion,
.

.
PP10. Prove that for any
there is the inequality
.
Proof. I"ll show that
. Indeed, using the substitution
the relation
becomes
. Using the substitution
obtain that
, what is truly. I have equality
if and only if
. In conclusion,
.
PP11. Find the maximum value of
.
Proof 1
. Let a mobile
and fixed
,
. Now
.
Have the equality for
. Hence
.
PP12. Prove that
.
Proof.
.
Remark. I used the remarkable power main inequality
.
PP13. Let
,
and
. Prove that
.
Proof.
and

. In conclusion,
.
PP14. Prove that for
,
.
Proof.
.
PP15. Let
. Prove that
.
Proof 1. Observe that
.
Proof 2 (ugly). Observe that

, what is truly. We have the equality if and only if
, i.e.
.
Proof 3.
. Therefore, 
, it is true. Have equality iff
, i.e.
.
An easy extension (sqing). Prove that for any
there is the inequality
.
Proof.
. Particular case. If
and
, then
.
PP16. Prove that the implication:
.
Proof 0 (with derivatives). Denote
. Prove easily that

and
. In conclusion
, i.e.
.
Proof 1.
is maximum
is maximum
is maximum

is maximum . Since
(constant) obtain that
is maximum

. With othet words,
.
Proof 2.
, where
. Thus, 
, what is truly. Have equality if and only if
.
An easy extension. Prove that the implication:
.


Proof 1.


Proof 2. Since





Can apply the Chebyshev's inequality so that

Similar proposed problem. Prove that


![$S=[ABC]$](http://latex.artofproblemsolving.com/b/3/a/b3ae3d445111e4dd28be75922309d3270079368c.png)

Proof. I"ll use the well-known identity













Otherwise.






Observe that






PP2. Prove that
![$\left\{a,b,c\right\}\subset (0,1]\ \implies\ a+b+c+ \frac{1}{abc} \ge\frac{1}{a} + \frac{1}{b} + \frac{1}{c} +abc$](http://latex.artofproblemsolving.com/2/3/8/2381578c8a0432cf3c4614e3773b3c6dc6631c51.png)
Proof. Denote
![$\left\{x,y,z\right\}\subset (0,1]$](http://latex.artofproblemsolving.com/e/2/c/e2cc3ea087e3966cf78b09974b99d9aed8db74b9.png)


becomes






An easy and nice extension. Prove that
![$(\forall )\ k\in\overline{1,n}\ ,\ x_k\in (0,1]\implies \sum_{k=1}^nx_k+$](http://latex.artofproblemsolving.com/c/3/4/c3480b40b04f75e91a508668f92f6075dd9621a5.png)


Proof. Denote

![$x\in (0,1]$](http://latex.artofproblemsolving.com/8/8/5/885ef0336d15614e1f1fcac9cf1aa48751c547e1.png)


Therefore,




PP3. Prove that for any point



Proof. Observe that


![$\sum a\cdot\left[PA+\delta_{BC}(P)\right]\ge\sum ah_a=6S\implies$](http://latex.artofproblemsolving.com/4/e/d/4ede78b2ecd6242f97f8474ffe7eac2d83770eb8.png)


PP4. Find the minimum value of

Proof. Denote





![$\frac 14\cdot\left[\left(4\cos a+\cos b\right)^2+\sin^2b-9\right]\ge -\frac 94$](http://latex.artofproblemsolving.com/8/c/0/8c0a37c495ae3d423d68d58ac377684dbd77ea69.png)
We have equality if and only if



PP5. Let



iff




Proof 1. If






















Remark. If






Proof 2. By squaring both sides of the given inequality one obtains:



is trivial by AM-GM inequality. Equality occurs iff








PP6. Let







that
![$R \leq \frac{a^2+b^2+c^2}{18\cdot\sqrt [3]{xyz}}$](http://latex.artofproblemsolving.com/d/7/a/d7abea8c2d3b36c689b7878a2dc2b19e488fec7f.png)
Proof.

![$3\cdot \sqrt[3]{abcxyz}\le 2S\ (*)$](http://latex.artofproblemsolving.com/2/8/9/289b8108da4a0374d180558c97f5438986fe4774.png)
![$3\cdot\sqrt [3]{a^2b^2c^2}\le \sum a^2\implies$](http://latex.artofproblemsolving.com/e/3/1/e3184bc4144168eb49b334befa2f84f89a289aa9.png)
![$12RS=3\cdot a^2b^2c^2\le\sqrt [3]{abc}\cdot\sum a^2\implies$](http://latex.artofproblemsolving.com/8/3/d/83d65e6ea4cf3a349a5730ebf1f43798557d789e.png)
![$12RS\cdot \sqrt [3]{xyz}\le \left(a^2+b^2+c^2\right)\cdot \sqrt[3]{abcxyz}\stackrel{(*)}{\le} \left(a^2+b^2+c^2\right)\cdot\frac {2S}{3}\implies$](http://latex.artofproblemsolving.com/8/f/b/8fbed2fcac2ab21aab5ae2eefeba4849b8864872.png)
![$18R\cdot\sqrt [3]{xyz}\le a^2+b^2+c^2$](http://latex.artofproblemsolving.com/9/5/a/95a1aa7b57a8b448b6d03a708d59003c305b0e1c.png)
PP7. Prove that

Proof 1. Minkowski's inequality :

![$ \ge \sqrt {{{\left( {\frac{{x - z}}{2}} \right)}^2} + {{\left[ {\frac{{\left( {x + z} \right)\sqrt 3 }}{2}} \right]}^2}} = \sqrt {{x^2} + xz + {z^2}}$](http://latex.artofproblemsolving.com/8/5/4/8546ae9d4721a5854151e53e089440f2a9bb2434.png)
Proof 2. Let






Observe that



in

![$[VB]$](http://latex.artofproblemsolving.com/8/8/3/883b7ed04e16efe64d1afe2067603b90849dab1f.png)










becomes







Proof 3.





An easy extension.




Proof. Let






![$\phi\in\left[0,\frac {\pi}{2}\right]$](http://latex.artofproblemsolving.com/8/7/6/876cea07f0de8be76457a92a1a237e10d79f07ff.png)
![$m=2\cos\phi\in [0,2]$](http://latex.artofproblemsolving.com/9/4/3/9431fe14c1d7c22b2fae6a41cedf222ddb575fdc.png)
Observe that



what is truly. We have equality


![$[VB]$](http://latex.artofproblemsolving.com/8/8/3/883b7ed04e16efe64d1afe2067603b90849dab1f.png)









PP8. Let


Proof.

An easy extension. Let



Proof.



PP9. Let

![$\left\{\begin{array}{cccc}
8(a+b+c)(ab+bc+ca) & \le & 9(b+c)(c+a)(a+b) & (1)\\\\
2\cdot \sqrt{bc+ca+ab} & \leq & \sqrt{3}\cdot \sqrt[3]{(b+c)(c+a)(a+b)} & (2)\\\\
8\sqrt {abc(a+b+c)(ab+bc+ca)} & \le & 3(b+c)(c+a)(a+b) & (3)\end{array}\right\|$](http://latex.artofproblemsolving.com/1/5/f/15f494781023880b600bd81056e6f75d86afb93d.png)
Proof. Denote


Observe that

![$ 3\sqrt [3]{abc}\cdot 3\sqrt [3]{a^2b^2c^2}=$](http://latex.artofproblemsolving.com/2/a/b/2ab3c3a5a385cadd38a23c73e6e25b85043c813e.png)

Therefore,




In conclusion,


![$\boxed{\sqrt{3}\cdot \sqrt[3]{(b+c)(c+a)(a+b)}\ge 2\cdot \sqrt{bc+ca+ab}}\ (2)$](http://latex.artofproblemsolving.com/1/9/4/194463afac4715caf3f0b3d635497f696f3f58c9.png)



PP10. Prove that for any


Proof. I"ll show that









if and only if



PP11. Find the maximum value of

Proof 1





Have the equality for



PP12. Prove that

Proof.


Remark. I used the remarkable power main inequality
![$\sqrt[2]{\frac {x^2+y^2+z^2}{3}}\le \sqrt [3]{\frac {x^3+y^3+z^3}{3}}$](http://latex.artofproblemsolving.com/7/6/a/76aa18772198f70ce0472106432053fb5e16586a.png)
PP13. Let




Proof.








PP14. Prove that for


Proof.



PP15. Let


Proof 1. Observe that




Proof 2 (ugly). Observe that









Proof 3.


![$4\cdot\sqrt[4]{\frac {3\sqrt3\left(\sin^2x+\cos^2x\right)^2}{\sin^3x\cos x}}$](http://latex.artofproblemsolving.com/c/8/2/c825c6aad1c7a9fe86dfd66829b77392625b13bb.png)

![$4\cdot\sqrt[4]{\frac {3\sqrt3\left(t^2+1\right)^2}{t^3}}$](http://latex.artofproblemsolving.com/4/3/9/439f0de599203a00ce6737a5e91833d20e52834c.png)






An easy extension (sqing). Prove that for any


Proof.





PP16. Prove that the implication:

Proof 0 (with derivatives). Denote




and





Proof 1.







is maximum . Since








Proof 2.










An easy extension. Prove that the implication:

This post has been edited 157 times. Last edited by Virgil Nicula, Nov 21, 2015, 5:43 PM